LSAT and Law School Admissions Forum

Get expert LSAT preparation and law school admissions advice from PowerScore Test Preparation.

 Administrator
PowerScore Staff
  • PowerScore Staff
  • Posts: 8929
  • Joined: Feb 02, 2011
|
#104141
Complete Question Explanation

Flaw in the Reasoning. The correct answer choice is (A).

Answer choice (A): This is the correct answer choice.

Answer choice (B):

Answer choice (C):

Answer choice (D):

Answer choice (E):

This explanation is still in progress. Please post any questions below!
User avatar
 npant120
  • Posts: 20
  • Joined: Aug 27, 2023
|
#105240
Hello,

I had a question on answer choice B as I noticed a lot of people selected this and I was also between A and B while taking the exam. Ultimately I chose A because I knew that it was a flaw of the argument whereas I was unsure of whether or not B occurred. What is confusing to me is that when looking at this problem, it seems that the conclusion is the necessary condition of the conditional chain created in the stimulus. Namely:
grow rapidly->innovative->creative employees or management open to new ideas.

The conclusion essentially is that the management must be open to new ideas but isn't it true that we cannot prove this? Unless we are assuming the sufficient condition of this is true? I'm not sure if my question makes sense but I guess I am just wondering what definitively makes B wrong, though I do understand that A is correct.
 Robert Carroll
PowerScore Staff
  • PowerScore Staff
  • Posts: 1787
  • Joined: Dec 06, 2013
|
#105246
Nimisha,

Answer choice (B) describes a Mistaken Reversal. Now, as you point out, we have conditional reasoning in this stimulus, but the flaw isn't Mistaken Reversal. In fact, the flaw isn't conditional at all. The conclusion is the necessary condition of the conditional. That already means the flaw can't be conditional - conditionals allow us to prove the necessary is true (when the argument adduces the sufficient condition as a premise) or that the sufficient condition is false (when the argument adduces the negation of the necessary condition as a premise). So if the conclusion is the necessary, that means there is a way for this argument to work conditionally. If it still has a mistake, it can't be a conditional error. Since that's what answer choice (B) says it is, that answer is incorrect.

Instead, focus on what you said: "The conclusion essentially is that the management must be open to new ideas but isn't it true that we cannot prove this? Unless we are assuming the sufficient condition of this is true?" Indeed, the conclusion could have been proven if the sufficient condition had been established. So what's the sufficient condition? "A company grows rapidly" is. We don't know that about Logichut. We DO know that about the industry. So the author has made the leap from "the industry is growing rapidly and has been" to "Logichut is growing rapidly and has been". If that leap were allowed, the conditional would WORK. So the error is in the leap only. The leap is a whole-to-part error, making answer choice (A) correct.

Robert Carroll
User avatar
 npant120
  • Posts: 20
  • Joined: Aug 27, 2023
|
#105250
Robert Carroll wrote: Fri Feb 09, 2024 6:42 am Nimisha,

Answer choice (B) describes a Mistaken Reversal. Now, as you point out, we have conditional reasoning in this stimulus, but the flaw isn't Mistaken Reversal. In fact, the flaw isn't conditional at all. The conclusion is the necessary condition of the conditional. That already means the flaw can't be conditional - conditionals allow us to prove the necessary is true (when the argument adduces the sufficient condition as a premise) or that the sufficient condition is false (when the argument adduces the negation of the necessary condition as a premise). So if the conclusion is the necessary, that means there is a way for this argument to work conditionally. If it still has a mistake, it can't be a conditional error. Since that's what answer choice (B) says it is, that answer is incorrect.

Instead, focus on what you said: "The conclusion essentially is that the management must be open to new ideas but isn't it true that we cannot prove this? Unless we are assuming the sufficient condition of this is true?" Indeed, the conclusion could have been proven if the sufficient condition had been established. So what's the sufficient condition? "A company grows rapidly" is. We don't know that about Logichut. We DO know that about the industry. So the author has made the leap from "the industry is growing rapidly and has been" to "Logichut is growing rapidly and has been". If that leap were allowed, the conditional would WORK. So the error is in the leap only. The leap is a whole-to-part error, making answer choice (A) correct.

Robert Carroll
Thank you! This was very helpful

Get the most out of your LSAT Prep Plus subscription.

Analyze and track your performance with our Testing and Analytics Package.